Add problems for Serway and Jewett v8's chapter 31.
[course.git] / latex / problems / Serway_and_Jewett_8 / problem31.23.tex
1 \begin{problem*}{31.23}
2 Figure~P31.23 shows a top view of a bar that can slide on two
3 frictionless rails.  The resistor is $R=6.00\U{\Ohm}$, and a
4 $2.50\U{T}$ magnetic field is directed perpendicularly downward, into
5 the paper.  Let $l=1.20\U{m}$.  \Part{a} Calculate the applied force
6 required to move the bar to the right at a constant speed of
7 $2.00\U{m/s}$.  \Part{b} At what rate is energy delivered to the
8 resistor?
9 \begin{center}
10 %   +-------+--------
11 %   Z x x^x | x x Bin
12 % R Z  l |  |--> F_app
13 %   Z x xvx | x x x x
14 %   +-------+--------
15 \begin{asy}
16 import Mechanics;
17 import ElectroMag;
18 import Circ;
19
20 real u = 1cm;
21 real w = 1mm;
22
23 MultiTerminal R = resistor(dir=90, "$R$", draw=false);
24 real rlen = R.terminal[1].y - R.terminal[0].y;
25 Vector B = BField(phi=-90);
26 vector_field(R.center + (0.75*u, 0), width=1.5*u, height=rlen, v=B);
27 R.draw();
28 real yt = R.terminal[1].y;
29 real yb = R.terminal[0].y;
30 wire((0, yt), (1.5*u, yt));
31 wire((0, yb), (1.5*u, yb));
32 pair p = (u,(yt+yb)/2);
33 Vector F = Force(p, Label("$F_\text{app}$", position=EndPoint));  F.draw();
34 Block block = Block(p, width=w, height=rlen);  block.draw();
35 Distance Dl = Distance((u/2, yt), (u/2, yb), Label("$l$", embed=Shift));
36 Dl.draw();
37 \end{asy}
38 \end{center}
39 \end{problem*}
40
41 \begin{solution}
42 \end{solution}
43